Difference between revisions of "2024 AMC 10A Problems/Problem 21"
(→Solution) |
Mathfun2012 (talk | contribs) m |
||
Line 5: | Line 5: | ||
==Solution== | ==Solution== | ||
− | + | <cmath> \begin{bmatrix} 12 & 29 &46&63&80 \\ 12&24&36&48&60\\ 12&19&26&33&40\\ 12&14&16&18&20\\ 12&9&6&3&0\end{bmatrix}</cmath> | |
− | + | -submitted by Astingo | |
− | |||
− | |||
− | |||
− | |||
− | |||
− | |||
− | |||
− | submitted by Astingo |
Revision as of 17:30, 8 November 2024
Problem
The numbers, in order, of each row and the numbers, in order, of each column of a array of integers form an arithmetic progression of length The numbers in positions and are and respectively. What number is in position
Solution
-submitted by Astingo